#1. Find the greatest number that will divide 43, 91 and 183 so as to leave the same remainder in each case.
A. 4
B. 7
C. 9
D. 13
Answer: Option A Solution: To find the greatest number that will divide 43, 91, and 183 while leaving the same remainder in each case, follow these steps:
Step 1: Find the differences between the numbers Since the remainder is the same in each case, subtracting one number from another will cancel out the remainder. Thus, we compute:
\[ 91 – 43 = 48 \]
\[ 183 – 91 = 92 \]
\[ 183 – 43 = 140 \]
Step 2: Find the greatest common divisor (GCD) of the differences We need to find the GCD of 48, 92, and 140.
2. Identify common factors: – The only common factor in all three numbers is \( 2^2 = 4 \).
Step 3: Conclusion The greatest number that will divide 43, 91, and 183 while leaving the same remainder in each case is 4.
#2. The H.C.F. of two numbers is 23 and the other two factors of their L.C.M. are 13 and 14. The larger of the two numbers is:
A. 276
B. 299
C. 322
D. 345
Answer: Option C Solution: We are given:
– The HCF (GCD) of two numbers = 23 – The other two factors of their LCM = 13 and 14
Step 1: Express the Relationship between HCF and LCM We use the fundamental relation:
\[ \text{Product of two numbers} = \text{HCF} \times \text{LCM} \]
Since the LCM contains the HCF as a factor, we express the LCM as:
\[ \text{LCM} = 23 \times 13 \times 14 \]
Step 2: Find the Two Numbers Let the two numbers be 23a and 23b, where a and b are co-prime (i.e., they have no common factors other than 1).
From the given data:
\[ \text{LCM} = 23 \times 13 \times 14 \]
Since LCM of 23a and 23b must be 23 × 13 × 14, the values of a and b must be 13 and 14.
Thus, the two numbers are:
\[ 23 \times 13 = 299 \] \[ 23 \times 14 = 322 \]
Step 3: Identify the Larger Number The larger of the two numbers is 322.
Final Answer: 322.
#3. Six bells commence tolling together and toll at intervals of 2, 4, 6, 8 10 and 12 seconds respectively. In 30 minutes, how many times do they toll together ?
A. 4
B. 10
C. 15
D. 16
Answer: Option D Solution: To determine how many times the six bells toll together in 30 minutes, follow these steps:
Step 1: Find the Least Common Multiple (LCM) of the Given Intervals The bells toll at intervals of 2, 4, 6, 8, 10, and 12 seconds. The first time they toll together is at time = 0 seconds. After that, they toll together every LCM of (2, 4, 6, 8, 10, 12) seconds.
2. Taking the highest powers of all prime factors: – Highest power of 2: \(2^3 = 8\) – Highest power of 3: \(3^1 = 3\) – Highest power of 5: \(5^1 = 5\)
Step 2: Find How Many Times They Toll Together in 30 Minutes – The first tolling happens at t = 0. – After that, they toll together every 120 seconds (2 minutes). – In 30 minutes, the number of times they toll together:
\[ \frac{30}{2} + 1 = 15 + 1 = 16 \]
Final Answer: 16 times.
#4. Let N be the greatest number that will divide 1305, 4665 and 6905, leaving the same remainder in each case. Then sum of the digits in N is:
A. 4
B. 5
C. 6
D. 8
Answer: Option A Solution: N = H.C.F. of (4665 – 1305), (6905 – 4665) and (6905 – 1305) = H.C.F. of 3360, 2240 and 5600 = 1120. Sum of digits in N = ( 1 + 1 + 2 + 0 ) = 4
#5. The greatest number of four digits which is divisible by 15, 25, 40 and 75 is:
A. 9000
B. 9400
C. 9600
D. 9800
Answer: Option C Solution: To find the greatest four-digit number that is divisible by 15, 25, 40, and 75, follow these steps:
Step 1: Find the LCM of 15, 25, 40, and 75 We first determine the least common multiple (LCM) of the given numbers.
Taking the highest powers of all prime factors: – Highest power of 2: \( 2^3 = 8 \) – Highest power of 3: \( 3^1 = 3 \) – Highest power of 5: \( 5^2 = 25 \)
\[ \text{LCM} = 8 \times 3 \times 25 = 600 \]
Step 2: Find the Greatest Four-Digit Number Divisible by 600 The greatest four-digit number is 9999. We need to find the largest multiple of 600 that is ≤ 9999.
Final Answer: The greatest four-digit number divisible by 15, 25, 40, and 75 is 9600. \[ \boxed{9600} \]
#6. The product of two numbers is 4107. If the H.C.F. of these numbers is 37, then the greater number is:
A. 101
B. 107
C. 111
D. 185
Answer: Option C Solution: Let the numbers be 37a and 37b Then, 37a x 37b = 4107 ⇒ ab = 3 Now, co-primes with product 3 are (1, 3) So, the required numbers are (37 x 1, 37 x 3) i.e., (37, 111) ∴ Greater number = 111
#7. Three number are in the ratio of 3 : 4 : 5 and their L.C.M. is 2400. Their H.C.F. is:
A. 40
B. 80
C. 120
D. 200
Answer: Option A Solution: We are given:
– Three numbers in the ratio 3:4:5. – Their LCM = 2400. – We need to find their HCF.
Step 1: Express the Numbers in Terms of a Common Factor Let the numbers be:
\[ 3x, 4x, 5x \]
where x is the common factor.
Step 2: Find the LCM in Terms of \( x \) The LCM of \( 3x, 4x, 5x \) is:
Step 3: Find the HCF Since the numbers are 3x, 4x, 5x, their HCF is simply \( x \), which is:
\[ \mathbf{40} \] Final Answer: \[ \boxed{40} \]
#8. The G.C.D. of 1.08, 0.36 and 0.9 is:
A. 0.03
B. 0.9
C. 0.18
D. 0.108
Answer: Option C Solution: To find the GCD (Greatest Common Divisor) of 1.08, 0.36, and 0.9, follow these steps:
Step 1: Convert to Whole Numbers Since all numbers have decimals, multiply each by 100 to convert them to whole numbers:
\[ 1.08 \times 100 = 108 \]
\[ 0.36 \times 100 = 36 \]
\[ 0.90 \times 100 = 90 \]
Now, we find the **GCD of 108, 36, and 90.
Step 2: Find the GCD of 108, 36, and 90 Prime factorizations: – \( 108 = 2^2 \times 3^3 \) – \( 36 = 2^2 \times 3^2 \) – \( 90 = 2^1 \times 3^2 \times 5 \)
Identify the common factors: – Smallest power of 2: \( 2^1 = 2 \) – Smallest power of 3: \( 3^2 = 9 \)
Thus, the GCD is:
\[ 2 \times 9 = 18 \]
Step 3: Convert Back to Decimal Form Since we originally multiplied by **100**, divide the result by 100:
\[ \frac{18}{100} = 0.18 \] Final Answer: \[ \boxed{0.18} \]
#9. The product of two numbers is 2028 and their H.C.F. is 13. The number of such pairs is:
A. 1
B. 2
C. 3
D. 4
Answer: Option B Solution: Let the numbers 13a and 13b Then, 13a x 13b = 2028 ⇒ ab = 12 Now, the co-primes with product 12 are (1, 12) and (3, 4) [Note: Two integers a and b are said to be co-prime or relatively prime if they have no common positive factor other than 1 or, equivalently, if their greatest common divisor is 1 ] So, the required numbers are (13 x 1, 13 x 12) and (13 x 3, 13 x 4) Clearly, there are 2 such pairs.
#10. The least multiple of 7, which leaves a remainder of 4, when divided by 6, 9, 15 and 18 is:
A. 74
B. 94
C. 184
D. 364
Answer: Option D Solution: To find the least multiple of 7 that leaves a remainder of 4 when divided by 6, 9, 15, and 18, follow these steps:
Step 1: Find the LCM of 6, 9, 15, and 18 First, determine the Least Common Multiple (LCM) of the given divisors.
Take the highest powers of each prime factor: – Highest power of 2: \( 2^1 = 2 \) – Highest power of 3: \( 3^2 = 9 \) – Highest power of 5: \( 5^1 = 5 \)
\[ \text{LCM} = 2 \times 9 \times 5 = 90 \]
Step 2: Find the Smallest Number of the Form \( 90k + 4 \) That is a Multiple of 7 We need to find the smallest integer \( k \) such that:
\[ 90k + 4 \equiv 0 \pmod{7} \]
or equivalently,
\[ 90k \equiv -4 \pmod{7} \]
Since \( 90 \equiv 6 \pmod{7} \), we substitute:
\[ 6k \equiv -4 \pmod{7} \]
Step 3: Solve for \( k \) Rewriting \(-4\) in modulo 7:
So, the lowest common multiple of 24, 36, and 40 is 360.
#12. The least number which should be added to 2497 so that the sum is exactly divisible by 5, 6, 4 and 3 is:
A. 3
B. 13
C. 23
D. 33
Answer: Option C Solution: To find the least number that should be added to 2497 so that the sum is divisible by 5, 6, 4, and 3, we need to determine the least common multiple (LCM) of 5, 6, 4, and 3 first.
Now, we need to find the remainder when 2497 is divided by 60:
\[ 2497 \div 60 = 41 \text{ remainder } 37 \]
To make the sum divisible by 60, we need to add the difference between 60 and the remainder:
\[ 60 – 37 = 23 \]
Therefore, the least number that should be added to 2497 is 23.
#13. Reduce \( \frac{128352}{238368} \) to its lowest terms.
A.\( \frac{2}{4} \)
B. \( \frac{5}{13} \)
C. \( \frac{7}{13} \)
D. \( \frac{9}{13} \)
Answer: Option C Solution To reduce the fraction \( \frac{128352}{238368} \) to its lowest terms, we need to find the greatest common divisor (GCD) of the numerator (128352) and the denominator (238368), and then divide both by the GCD.
Let’s find the GCD of 128352 and 238368.
The greatest common divisor (GCD) of 128352 and 238368 is 18336.
Now, when we divide both the numerator and denominator by 18336, the fraction \( \frac{128352}{238368} \) reduces to:
\[ \frac{7}{13} \] So, \( \frac{128352}{238368} \) in its lowest terms is \( \frac{7}{13} \).
#14. The least number which when divided by 5, 6 , 7 and 8 leaves a remainder 3, but when divided by 9 leaves no remainder, is:
A. 1677
B. 1683
C. 2523
D. 3363
Answer: Option B Solution: To solve this problem, we need to find the smallest number \( x \) that satisfies the following conditions:
1. When \( x \) is divided by 5, 6, 7, and 8, it leaves a remainder of 3. 2. When \( x \) is divided by 9, it leaves no remainder.
Step 1: Translate the conditions into mathematical equations.
– The first condition can be written as: \[ x \equiv 3 \pmod{5}, \quad x \equiv 3 \pmod{6}, \quad x \equiv 3 \pmod{7}, \quad x \equiv 3 \pmod{8} \] This means that \( x – 3 \) is divisible by 5, 6, 7, and 8. So, \( x – 3 \) is a multiple of the least common multiple (LCM) of 5, 6, 7, and 8.
The LCM is the highest power of each prime: \[ LCM = 2^3 \times 3 \times 5 \times 7 = 8 \times 3 \times 5 \times 7 = 840 \] So, \( x – 3 \) must be a multiple of 840. Therefore, we can express \( x \) as: \[ x = 840k + 3 \] where \( k \) is some integer.
Step 3: Use the second condition.
The second condition states that \( x \) must be divisible by 9: \[ x \equiv 0 \pmod{9} \] Substitute \( x = 840k + 3 \) into this condition: \[ 840k + 3 \equiv 0 \pmod{9} \] This simplifies to: \[ 840k \equiv -3 \pmod{9} \] Now, reduce 840 modulo 9: \[ 840 \div 9 = 93 \text{ remainder } 3 \] So, \( 840 \equiv 3 \pmod{9} \). Therefore: \[ 3k \equiv -3 \pmod{9} \] This simplifies to: \[ k \equiv -1 \pmod{3} \] which is equivalent to: \[ k \equiv 2 \pmod{3} \]
Step 4: Find the smallest \( k \).
The smallest value of \( k \) that satisfies \( k \equiv 2 \pmod{3} \) is \( k = 2 \).
Step 5: Find the smallest \( x \).
Now, substitute \( k = 2 \) into \( x = 840k + 3 \): \[ x = 840(2) + 3 = 1680 + 3 = 1683 \]
Thus, the least number that satisfies all the conditions is 1683.
#15. A, B and C start at the same time in the same direction to run around a circular stadium. A completes a round in 252 seconds, B in 308 seconds and c in 198 seconds, all starting at the same point. After what time will they again at the starting point ?
A. 26 minutes and 18 seconds
B. 42 minutes and 36 seconds
C. 45 minutes
D. 46 minutes and 12 seconds
Answer: Option D Solution: L.C.M. of 252, 308 and 198 = 2772 So, A, B and C will again meet at the starting point in 2772 sec. i.e., 46 min. 12 sec.
#16. The H.C.F. of two numbers is 11 and their L.C.M. is 7700. If one of the numbers is 275, then the other is:
A. 279
B. 283
C. 308
D. 318
Answer: Option C Solution: We can use the following relationship between the HCF (Highest Common Factor), LCM (Least Common Multiple), and the two numbers:
Let the other number be \( x \). Using the formula:
\[ 11 \times 7700 = 275 \times x \]
Now, solve for \( x \):
\[ x = \frac{11 \times 7700}{275} \] Let’s calculate that. The other number is 308.
#17. What will be the least number which when doubled will be exactly divisible by 12, 18, 21 and 30 ?
A. 196
B. 630
C. 1260
D. 2520
Answer: Option B Solution: To find the least number \( x \) such that when doubled, it is divisible by 12, 18, 21, and 30, we need to follow these steps:
Step 1: Find the least common multiple (LCM) of 12, 18, 21, and 30.
We first find the prime factorizations of the numbers:
We divided 252 by 2 twice, by 3 twice, and ended with 7. So, the prime factorization of 252 is: \[ 252 = 2^2 \times 3^2 \times 7 \] Thus, 252 can be expressed as the product of primes \( 2^2 \times 3^2 \times 7 \).
#21. The greatest possible length which can be used to measure exactly the lengths 7 m, 3 m 85 cm, 12 m 95 cm is:
A. 15 cm
B. 25 cm
C. 35 cm
D. 42 cm
Answer: Option C Solution: To find the greatest possible length that can be used to measure exactly 7 meters, 3 meters 85 cm, and 12 meters 95 cm, we need to calculate the greatest common divisor (GCD) of the three lengths. First, we need to express all the lengths in the same unit, say in centimeters.
So, the GCD of 35 and 1295 is 35. Conclusion: The greatest possible length that can be used to measure exactly the lengths 7 m, 3 m 85 cm, and 12 m 95 cm is 35 cm.
#22. Three numbers which are co-prime to each other are such that the product of the first two is 551 and that of the last two is 1073. The sum of the three numbers is:
A. 75
B. 81
C. 85
D. 89
Answer: Option C Solution Let the three numbers be \( a \), \( b \), and \( c \), and we are given the following information:
– The product of the first two numbers: \( a \times b = 551 \) – The product of the last two numbers: \( b \times c = 1073 \)
Also, the three numbers are co-prime to each other, meaning that their pairwise greatest common divisor (GCD) is 1.
Step 1: Find the common factor \( b \)
We know that \( a \times b = 551 \) and \( b \times c = 1073 \), so \( b \) must be a common factor of 551 and 1073.
We will find the GCD of 551 and 1073 using the Euclidean algorithm:
The common prime factors between 36 and 84 are: – The highest power of 2 is \( 2^2 \). – The highest power of 3 is \( 3 \).
Step 3: Multiply the common factors.
The HCF is the product of the common factors: \[ HCF = 2^2 \times 3 = 4 \times 3 = 12 \] Conclusion: The highest common factor (HCF) of 36 and 84 is 12.
#24. Which of the following fraction is the largest ?
A. \( \frac{7}{8} \)
B. \( \frac{13}{16} \)
C. \( \frac{31}{40} \)
D. \( \frac{63}{80} \)
Answer: Option A Solution: To determine which of the fractions \( \frac{7}{8} \), \( \frac{13}{16} \), \( \frac{31}{40} \), and \( \frac{63}{80} \) is the largest, we can compare them by converting each fraction to a decimal or by finding a common denominator. Let’s convert them to decimal values:
Since the remainder is 0, the GCD of 1651 and 2032 is 127. Conclusion: The greatest number that, when dividing 1657 and 2037, leaves remainders 6 and 5, respectively, is 127.
#27. Which of the following has the most number of divisors?
A. 99
B. 101
C. 176
D. 182
Answer: Option C Solution: 99 = 1 × 3 × 3 × 11 101 = 1 × 101 176 = 1 × 2 × 2 × 2 × 2 × 11 182 = 1 × 2 × 7 × 13 So, divisors of 99 are 1, 3, 9, 11, 33, 99 Divisors of 101 are 1 and 101 Divisors of 176 are 1, 2, 4, 8, 11, 16, 22, 44, 88 and 176 Divisors of 182 are 1, 2, 7, 13, 14, 26, 91 and 182 Hence, 176 has the most number of divisors.
#28. The L.C.M. of two numbers is 48. The numbers are in the ratio 2 : 3. Then sum of the number is:
A. 28
B. 32
C. 40
D. 64
Answer: Option C Solution: We are given that the LCM of two numbers is 48 and that the numbers are in the ratio 2:3. Let’s solve for the numbers and their sum.
Step 1: Represent the numbers in terms of a common variable.
Let the two numbers be \( 2x \) and \( 3x \), where \( x \) is a common factor.
Step 2: Use the LCM formula.
The LCM of two numbers is given by:
\[ \text{LCM} = \frac{\text{Product of the numbers}}{\text{GCD of the numbers}} \]
The GCD of \( 2x \) and \( 3x \) is \( x \) because the only common factor between 2 and 3 is 1, and both numbers share \( x \) as a common factor.
\[ 16 + 24 = 40 \] Conclusion: The sum of the two numbers is 40.
#29. The H.C.F. of \( \frac{9}{10} \),\( \frac{12}{25} \),\( \frac{18}{35} \) and \( \frac{21}{40} \)is:
A. \( \frac{3}{5} \)
B. \( \frac{252}{5} \)
C. \( \frac{3}{1400} \)
D. \( \frac{63}{700} \)
Answer: Option C Solution: To find the Highest Common Factor (HCF) of the fractions \( \frac{9}{10}, \frac{12}{25}, \frac{18}{35}, \) and \( \frac{21}{40} \), we will find the HCF of the numerators and the Least Common Multiple (LCM) of the denominators.
Step 1: Find the HCF of the numerators. The numerators are 9, 12, 18, and 21. Let’s find the HCF of these numbers:
To find the LCM, take the highest power of each prime factor:
– The highest power of 2 is \( 2^3 \) – The highest power of 5 is \( 5^2 \) – The highest power of 7 is \( 7 \)
Thus, the LCM of the denominators is:
\[ \text{LCM of the denominators} = 2^3 \times 5^2 \times 7 = 8 \times 25 \times 7 = 1400 \]
Step 3: Write the HCF of the fractions.
The HCF of the fractions is:
\[ \text{HCF} = \frac{\text{HCF of numerators}}{\text{LCM of denominators}} = \frac{3}{1400} \] Conclusion: The HCF of \( \frac{9}{10}, \frac{12}{25}, \frac{18}{35}, \) and \( \frac{21}{40} \) is \( \frac{3}{1400} \).
#30. If the sum of two numbers is 55 and the H.C.F. and L.C.M. of these numbers are 5 and 120 respectively, then the sum of the reciprocals of the numbers is equal to:
A.\( \frac{55}{601} \)
B.\( \frac{601}{55} \)
C.\( \frac{11}{120} \)
D.\( \frac{120}{11} \)
Answer: Option C Solution: We are given the following:
– The sum of two numbers \( a \) and \( b \) is 55: \( a + b = 55 \). – The HCF of \( a \) and \( b \) is 5. – The LCM of \( a \) and \( b \) is 120.
We are asked to find the sum of the reciprocals of the numbers, i.e.,
\[ \frac{1}{a} + \frac{1}{b} \]
Step 1: Use the relationship between HCF, LCM, and the product of two numbers.
We know the formula for the product of two numbers in terms of their HCF and LCM:
\[ \text{HCF}(a, b) \times \text{LCM}(a, b) = a \times b \]
Substituting the given values:
\[ 5 \times 120 = a \times b \quad \Rightarrow \quad a \times b = 600 \]
Given: – LCM = 495 – HCF = 5 – Sum of numbers = 100
Let the two numbers be x and y. Using the formula:
\[ 5 \times 495 = x \times y \]
\[ 2475 = x \times y \]
We also know:
\[ x + y = 100 \]
Step 1: Solve for \( x \) and \( y \) We solve the quadratic equation:
\[ t^2 – 100t + 2475 = 0 \]
Using the quadratic formula:
\[ t = \frac{-(-100) \pm \sqrt{(-100)^2 – 4(2475)}}{2} \]
\[ t = \frac{100 \pm \sqrt{10000 – 9900}}{2} \]
\[ t = \frac{100 \pm \sqrt{100}}{2} \]
\[ t = \frac{100 \pm 10}{2} \]
\[ t = \frac{110}{2} = 55 \quad \text{or} \quad t = \frac{90}{2} = 45 \]
So, the two numbers are 55 and 45.
Step 2: Find the Difference \[ 55 – 45 = 10 \] Thus, the difference between the two numbers is 10.
#34. Which of the following has most number of divisors?
A. 99
B. 101
C. 176
D. 182
Answer: Option C Solution: 99 = 1 × 3 × 3 × 11 101 = 1 × 101 176 = 1 × 2 × 2 × 2 × 2 × 11 182 = 1 × 2 × 7 × 13 So, divisor of 99 are 1, 3, 9, 11, 33 and 99 Divisor of 101 are 1 and 101 Divisor of 176 are 1, 2, 4, 8, 11, 16, 22, 44, 88 and 176 Divisor of 182 are 1, 2, 7, 13, 14, 26, 91 and 182 Hence, 176 has the most number of divisors.
#35. The HCF of two numbers is 8. Which one of the following can never be their LCM ?
A. 24
B. 48
C. 56
D. 60
Answer: Option D Solution: HCF = 8 Now LCM should have a factors 8 So check also the option we have only 60 which does not have a factor 8. So it will never be the LCM.
#36. The LCM and the HCF of the numbers 28 and 42 are in the ratio ?
A. 6 : 1
B. 2 : 3
C. 3 : 2
D. 7 : 2
Answer: Option A Solution: To find the ratio of the LCM and HCF of 28 and 42, we first determine their HCF and LCM.
Step 1: Find HCF of 28 and 42 The prime factorizations are:
Step 2: Find LCM of 28 and 42 The LCM takes the highest power of each prime factor:
\[ \text{LCM} = 2^2 \times 3^1 \times 7^1 = 84 \]
Step 3: Find the Ratio \[ \frac{\text{LCM}}{\text{HCF}} = \frac{84}{14} = 6:1 \] Final Answer: The ratio of LCM to HCF is 6:1.
#37. The simplest reduction to the lowest terms of \( \frac{116690151}{427863887} \) is = ?
A. \( \frac{3}{11} \)
B. \( \frac{7}{11} \)
C. \( \frac{11}{3} \)
D. None of these
Answer: Option A Solution: To simplify the fraction \( \frac{116690151}{427863887} \), we need to find the greatest common divisor (GCD) of the numerator and denominator and divide both by this value. Let’s compute the GCD of 116690151 and 427863887. The simplest form of \( \frac{116690151}{427863887} \) is \( \frac{3}{11} \).
#38. The LCM of two numbers is 1820 and their HCF is 26. If one number is 130 then the other number is ?
A. 70
B. 1690
C. 364
D. 1264
Answer: Option C Solution: We use the relationship between HCF, LCM, and the two numbers:
– The LCM takes the highest powers of all prime factors: – Highest power of 2: \( 2^3 = 8 \) – Highest power of 3: \( 3^1 = 3 \) – Highest power of 5: \( 5^1 = 5 \)
\[ \text{LCM} = 8 \times 3 \times 5 = 120 \]
Step 2: Find the Largest Five-Digit Number The largest five-digit number is 99999. We now find the largest multiple of 120 less than or equal to 99999.
\[ \frac{99999}{120} \approx 833.325 \]
So, the largest multiple of 120 is:
\[ 120 \times 833 = 99960 \]
Step 3: Add the Remainder Since the required number leaves a remainder of 2, we add 2: \[ 99960 + 2 = 99962 \] Final Answer: The greatest five-digit number that satisfies the given condition is 99962.
#41. The H.C.F of 24 × 32 × 53 × 7, 23 × 33 × 52 × 72 and 3 × 5 × 7 × 11 is = ?
A. 105
B. 1155
C. 2310
D. 27720
Answer: Option A Solution: To find the HCF (Highest Common Factor) of the three numbers, we need to take the lowest powers of the common prime factors present in each number.
Step 1: Find the Common Prime Factors The prime factors in these numbers are 2, 3, 5, 7, and 11.
We now look for the minimum power of each prime factor common to all three numbers:
– For 2: The first number has \( 2^4 \), the second has \( 2^3 \), and the third has no factor of \( 2 \). So, the minimum power of 2 is \( 2^0 \) (no factor). – For 3: The first number has \( 3^2 \), the second has \( 3^3 \), and the third has \( 3^1 \). So, the minimum power of 3 is \( 3^1 \). – For 5: The first number has \( 5^3 \), the second has \( 5^2 \), and the third has \( 5^1 \). So, the minimum power of 5 is \( 5^1 \). – For 7: The first number has \( 7^1 \), the second has \( 7^2 \), and the third has \( 7^1 \). So, the minimum power of 7 is \( 7^1 \). – For 11: The first and second numbers have no factor of \( 11 \), but the third has \( 11^1 \). So, the minimum power of 11 is \( 11^0 \) (no factor).
Step 2: Compute the HCF The HCF is the product of the common prime factors with the lowest powers:
\[ \text{HCF} = 3^1 \times 5^1 \times 7^1 = 3 \times 5 \times 7 = 105 \] Final Answer: The HCF of the three numbers is 105.
#42. The H.C.F of 4 × 27 × 3125, 8 × 9 × 25 × 7 and 16 × 81 × 5 × 11 × 49 is = ?
A. 180
B. 360
C. 540
D. 1260
Answer: Option D Solution: 42. The H.C.F of 4 × 27 × 3125, 8 × 9 × 25 × 7 and 16 × We are given three numbers:
Step 2: Find the Common Prime Factors Now, we compare the prime factorizations of all three numbers:
– For 2: The exponents are \( 2^2, 2^3, 2^4 \). The minimum exponent is \( 2^2 \). – For 3: The exponents are \( 3^3, 3^2, 3^4 \). The minimum exponent is \( 3^2 \). – For 5: The exponents are \( 5^5, 5^2, 5^1 \). The minimum exponent is \( 5^1 \). – For 7: The exponents are \( 7^1, 7^1, 7^2 \). The minimum exponent is \( 7^1 \). – For 11: The exponents are \( 11^0, 11^1, 11^1 \). The minimum exponent is \( 11^0 \) (no factor in the first number).
Step 3: Compute the HCF The HCF is the product of the common prime factors with their lowest exponents:
\[ \text{HCF} = 4 \times 9 \times 5 \times 7 = 36 \times 5 \times 7 = 180 \times 7 = 1260 \] Final Answer: The HCF of the three numbers is 1260.81 × 5 × 11 × 49 is = ?
#43. Even numbers are formed by talking at least two at a time from the numbers 0, 4, 8, 9. Their H.C.F. is = ?
A. 2
B. 4
C. 10
D. None fo these
Answer: Option A Solution: Since all the numbers formed are even, 2 is a common factor. Also, H.C.F. of two of the numbers i.e., 48 and 490, is 2. So, the H.C.F. of all the numbers formed is 2.
#44. Find the largest number of four digits such that on dividing by 15, 18, 21 and 24 the remainders are 11, 14, 17 and 20 respectively.
A. 6557
B. 7556
C. 5675
D. 7664
Answer: Option B Solution: LCM of (15, 18, 21, 24) ⇒ 5 × 3 × 6 × 7 × 4 = 2520 ⇒ In such type of questions,we take the difference between given number and remainder of that number. Number Remainder ⇒ (15 – 11) = 4 ⇒ (18 – 14) = 4 ⇒ (21 – 17) = 4 ⇒ (24 – 20) = 4 It will be same always Now, Largest 4 digit number is 9999 ⇒ On dividing 9999 by LCM (2520) we get remainder ⇒ 2439 Subtract remainder from 9999 we get largest 4 digit number, which is divisible by given number = 9999 – 2439 = 7560 (9999 but required number gives difference on dividing) So, our required number = 7560 – 4(difference) = 7556
#45. The greatest number which when divides 989 and 1327 leave remainder 5 and 7 respectively = ?
A. 8
B. 16
C. 24
D. 32
Answer: Option C Solution: To solve this problem, we need to find the greatest number that divides 989 and 1327 leaving remainders 5 and 7, respectively.
Steps: 1. Subtract the remainder from both numbers: – For 989, subtract 5: \( 989 – 5 = 984 \) – For 1327, subtract 7: \( 1327 – 7 = 1320 \)
2. Now, we need to find the greatest common divisor (GCD) of 984 and 1320, because the number we’re looking for must divide both of these numbers. We can calculate the GCD of 984 and 1320 using the Euclidean algorithm. Let me calculate that. The greatest number that divides both 989 and 1327, leaving remainders 5 and 7 respectively, is 24.
#46. A milkman has 75 litres milk in one cane and 45 litres in another. The maximum capacity of container which can measure milk of either container exact number ?
A. 1 litre
B. 5 litres
C. 15 litres
D. 25 litres
Answer: Option C Solution: To find the maximum capacity of the container that can measure the exact amount of milk from either of the containers, we need to find the Greatest Common Divisor (GCD) of 75 liters and 45 liters. The GCD represents the largest number that divides both 75 and 45 exactly. Let me calculate the GCD for you. The maximum capacity of the container that can measure the exact amount of milk from either of the containers is 15 liters.
#47. Which of the following is a pair of co-primes ?
A. (16, 12)
B. (18, 25)
C. (21, 35)
D. (23, 92)
Answer: Option B Solution: H.C.F. of 18 and 25 is 1 So, they are co-primes
#48. The L.C.M. of 22, 54, 108, 135 and 198 is = ?
A. 330
B. 1980
C. 5940
D. 11880
Answer: Option C Solution: To find the Least Common Multiple (LCM) of 22, 54, 108, 135, and 198, we need to find the smallest number that is divisible by all of them.
The LCM can be found by: 1. Factorizing each number into prime factors. 2. Taking the highest power of each prime factor that appears. Let me calculate the LCM for you. The Least Common Multiple (LCM) of 22, 54, 108, 135, and 198 is 5940.
#49. Two numbers are in the ratio 3 : 4, if their H.C.F. is 4, then their L.C.M. = ?
A. 48
B. 42
C. 36
D. 24
Answer: Option A Solution: When two numbers are in a given ratio and their HCF is known, the relationship between the HCF, LCM, and the numbers can be used to find the LCM. Here’s how:
Given: – The ratio of the two numbers is \( 3 : 4 \). – Their HCF is \( 4 \).
Let the numbers be \( 3x \) and \( 4x \), where \( x \) is a common factor. Since their HCF is \( 4 \), we know that \( x = 4 \).
So, the two numbers are: – \( 3x = 3 \times 4 = 12 \) – \( 4x = 4 \times 4 = 16 \)
Now, the formula for the product of two numbers is: \[ \text{HCF} \times \text{LCM} = \text{Product of the two numbers} \] Thus: \[ 4 \times \text{LCM} = 12 \times 16 \] Let me calculate the LCM using this formula. The Least Common Multiple (LCM) of the two numbers is 48.
#50. L.C.M. of two numbers is 2079 and their H.C.F. is 27. If one of the number is 189, the other number is = ?
A. 297
B. 584
C. 189
D. 216
Answer: Option A Solution: We can use the relationship between the LCM, HCF, and the two numbers to find the other number.
The formula is: \[ \text{LCM} \times \text{HCF} = \text{Product of the two numbers} \]
Given: – LCM = 2079 – HCF = 27 – One number = 189
Let the other number be \( x \). Then: \[ 2079 \times 27 = 189 \times x \] We can solve for \( x \). Let me calculate that for you. The other number is 297.